which of the variables listed above would best be displayed graphically using a bar graph? mark all that apply. group of answer choices number of minutes of exposure to secondhand smoke per day currently diagnosed with asthma race years of education gender

Answers

Answer 1

The two possible variables design are; "COHORT STUDY" and "CROSS-SECTIONAL STUDY".

Cohort study design is an observational study in which people who receives a particular treatment or are under a particular condition, are been observed and studied. This type of research model also allows a researcher to compare people affected by a condition to people unaffected by the same conditions.

The investigator is likely to use the cohort design because it will help in studying the children that are always exposed to smoke, and to observe the affected and unaffected children.

The CROSS-SECTIONAL design is an observational study in which a specific population are studied at a particular time. This type of research allows the researcher to study the difference in that population at a time interval.

This study design is likely to be used by the investigator, because it will help the investigator to determine the amount of secondhand smoke each child inhale at a particular time. It will also help the investigator to know the outcome of each child after 5years

learn more about of variable here

https://brainly.com/question/14861198

#SPJ4


Related Questions

A horse owner needs to replace four sections of a coral for the horses. The first section
needs 26 yards of fencing, the second needs 10 3/5, the third, 10 1/5 yards, and the fourth,
8 3/10 yards. How many yards are needed to repair four sections of the fencing?
10

Answers

Answer:

55 1/10 yd

Step-by-step explanation:

26 + 10 + 3/5 + 10 + 1/5 + 8 + 3/10 =

= 54 + 4/5 + 3/10 =

= 54 + (8 + 3)/10 =

= 54 + 11/10 =

= 54 + 10/10 + 1/10 =

= 54 + 1 + 1/10 =

= 55 1/10

Answer:

55[tex]\frac{1}{10}[/tex]

Step-by-step explanation:

Convert all the mixed numbers to fractions with a common denominator (10) then add them up:

26 + 53/5 + 51/5 + 83/10 =

26 + 106/10 + 102/10 + 83/10 =

26 + 291/10 =

26 + 29[tex]\frac{1}{10}[/tex] = 55[tex]\frac{1}{10}[/tex]

Can someone help please? Picture is already attached. If its wrong please correct it. This isn’t mine by the way, I’m posting it for someone else to double check if it’s right

Answers

Answer:

You've got it right

Step-by-step explanation:

If you want to get stellar communication on the question then maybe include a number line to visually represent why, but otherwise you have the answer right.

Can someone explain what the rules are for division algorithm and what it means?

Answers

Answer:

The division algorithm says when a number is divided by a number gives the quotient

Step-by-step explanation:

Find the constant term in the expansion of (2x+1/x)8

Answers

Answer:

the constant term in this problem is 8

I’m pretty sure it’s 8 !!

For which values of x does each expression make sense?
Square root of x+5
Square root of |x|+1
Square root of (-2x)^2
Square root of (-5x)^3

Answers

The value of x that makes sense for the expression is the domain of the expression
The value of x makes sense for √-2x² is 0
The value of x must not exceed 0 for  √(-2x)³ to make sense

What is Angle Sum Property?

The sum of all angles of a triangle is equal to the angle of a straight line i.e. 180°. If we have a triangle ABC, then the Sum of angles A , B, and angle C will be 180 ° and the value of the exterior angle is equal to the sum of two interior opposite angles.

√-2x²

For the above expression to have a defined value, the expression in the bracket must be positive.

However, x² will always be positive, and -2 will ensure that the expression is always negative, except when x = 0

Hence, the value of x makes sense for √-2x² is 0.

√(-2x)³

For the above expression to have a defined value, the expression in the square root must be positive.

However, the power of 3 will ensure that the expression is always positive only when x is less than or equal to 0

Hence, the value of x must not exceed 0 for √(-2x)³ to make sense.

To know more about the Angle sum property visit,

brainly.com/question/21099419

#SPJ1

using the following uniform density curve, determine what is the probability that a random variable has a value less than 44?

Answers

The probability that a random variable has a value less than 44 is 44.

The probability that a random variable has a value less than 44 can be determined by calculating the area under the curve to the left of 44.

In a uniform density curve, the probability of a random variable falling within a certain range is equal to the length of that range. To find the probability of a random variable having a value less than 44, we need to find the length of the range from 0 to 44.

The length of this range is 44 - 0 = 44.

Therefore, the probability that a random variable has a value less than 44 is 44.

Learn more about the probability at

https://brainly.com/question/29405460?referrer=searchResults

#SPJ4

Allie had four and three-fourths tubes of purple paint. She used two and one-fourth tubes. Later, she found three and three-eighths more tubes. How many tubes of purple paint does she have now?


five and thirteen over twenty-four tubes

five and nine over twenty-four tubes

five and seven over eight tubes

five and one over eight tubes

Answers

By subtracting and adding mixed numbers, we can see that at the end she has (5 + 7/8) tubes of paint.

How many tubes of purple paint does she have now?

We know that initially Allie had (4 + 3/4) tubes of purple paint, and she sed (2+ 1/4) tubes.

So at this point we need to take the difference between the two mixed numbers, we will get:

(4 + 3/4) - (2 + 1/4) = (4 - 2) + (3/4 - 1/4)

(4 - 2) + (3/4 - 1/4) = 2 + 2/4

2 + 2/4  = 2 + 1/2

Then she founds (3 + 3/8) more. Now we need to add that:

2 + 1/2 + (3 + 3/8) = (2 + 3) + (1/2 + 3/8)

(2 + 3) + (1/2 + 3/8) = 5 + (4/8 + 3/8)

5 + (4/8 + 3/8) = 5 + 7/8

She has 5 + 7/8 tubes of purple paint.

Learn more about mixed numbers:

https://brainly.com/question/21610929

#SPJ1

A systems analyst is testing the feasibility of using a new computer system. He wants to see if the new system uses less processing time than the old system. (Assume the original populations are normally distributed.) A sample of 25 jobs was selected and the processing time for each in seconds was recorded on each of the two systems. The results are as follows:Old System: mean = 27.2 seconds, s = 3.2 seconds, n - 25New System: mean = 24.3 seconds, s = 2.1 seconds, n = 25Difference (Old -New): mean = 2.9 seconds, s = 1.4 seconds, n = 25What is the alternative hypothesis for this problem?a. HA: µD≠0b. HA: µD>0c. HA: µD=0d. HA: µD<0

Answers

The alternative hypothesis for this problem b. HA: µD>0

What is meant by hypothesis?

In math, A hypothesis is an assumption made based on some evidence.

Here we have given that systems analyst is testing the feasibility of using a new computer system. He wants to see if the new system uses less processing time than the old system.

And we need to find the alternative hypothesis for this problem

While we looking into the given question we have identified the following values,

Old System: mean = 27.2 seconds, s = 3.2 seconds, n - 25

New System: mean = 24.3 seconds, s = 2.1 seconds, n = 25

Difference (Old -New): mean = 2.9 seconds, s = 1.4 seconds, n = 25

Basically, while written the hypothesis, we have to follow the order of the following,

=> (Old-New, Larger-Smaller>)

Therefore, the correct option is (B).

To know more about Hypothesis here.

https://brainly.com/question/29576929

#SPJ4

Determine the coordinates of the point shown.
Answer ASAP

A coordinate grid shown. There are increments of 0.5 for each grid line on each of the two axes. A point is located at 3 grid lines to the right and 1 grid line down from the origin.

(4 points)


(1.5, −0.5)

(−0.5, 1.5)

(3, −1)

(1, −1)

Answers

Answer:

(1.5, −0.5)

Step-by-step explanation:

A researcher was interested in comparing the resting pulse rates of people who exercise regularly and people who do not exercise regularly. Independent simple random samples were obtained of 16 people who do not exercise regularly and 12 people who do exercise regularly. The resting pulse rate (in beats per minute) of each person was recorded. The summary statistics are as follows.
Do Not Exercise - Do Exercise
x1 = 73.4 beats/min - x2 = 69.7 beats/min
s1 = 10.3 beats/min - s2 = 8.6 beats/min
n1 = 16 - n2 = 12
test the claim that he difference between the mean pulse rate of people who do not exercise regularly is higher than the mean pulse rate of people who exercise regularly.

Answers

We Reject H₀ if t calculated > t tabulated and hence the mean pulse rate of people who do not exercise regularly is higher than the mean pulse rate of people who exercise regularly

We Reject H₀ if t calculated > t tabulated

But in this case,

0.83 is not greater than 2.056

Therefore, we failed to reject H₀

There is no difference between the mean pulse rate of people who do not exercise and the mean pulse rate of people who do exercise

The Null and Alternate hypothesis is given by

Null hypotheses = H₀: μ₁ = μ₂

Alternate hypotheses = H₁: μ₁ ≠ μ₂

The test statistic is given by

Where  is the sample mean of people who do not exercise regularly.

Where  is the sample mean of people who do exercise regularly.

Where  is the sample standard deviation of people who do not exercise regularly.

Where  is the sample standard deviation of people who do exercise regularly.

Where  is the sample size of people who do not exercise regularly.

Where  is the sample size of people who do exercise regularly.

The given level of significance is

1 - 0.95 = 0.05

The degree of freedom is

df = 16 + 12 - 2 = 26

From the t-table, df = 26 and significance level 0.05,

t = 2.056 (two-tailed)

Conclusion:

We Reject H₀ if t calculated > t tabulated

learn more about of exercise here

https://brainly.com/question/17089447

#SPJ4

Which ordered pair is a solution to the system of inequalities shown on the graph?

A)(3, 3)

B) (1,2)

C)(-3,-2)

D) (0, 1)

Answers

Answer:

C)   (-3,-2)

Step-by-step explanation:

Solutions to the graphed system of inequalities are any points that are contained within the overlapping shaded region.

Plot the points on the given coordinate grid.

The only point that is contained with the overlapping shaded region is:

(-3, -2)

The boxplot below displays the average number of apple pies consumed in eating contests across a particular state. The higher outlier is from a professional, heavyweight contests, and the lower outlier is from a children's eating contest. X TI X 10 20 30 30 30 What effect will removing all outliers have on the mean and median of the data set? O a Neither the mean nor the median we change O b The mean will remain unchanged and the median will increase O c The mean will remain unchanged and the median wit decrease O d The median will remain unchanged and the mean will increase O e The median will remain unchanged and the mean will decat

Answers

The effect that will removing all outliers have on the mean and median of the data set is; C. median will remain unchanged and the mean will increase.

What are mean and median?

The mean is the average value which can be calculated by dividing the sum of observations by the number of observations

Mean = Sum of observations/the number of observations

The Median represents the middle value of the given data when arranged in a particular order.

WE have been given boxplot that displays the average number of apple pies consumed in eating contests across a particular state.

The higher outlier is from a professional,the heavyweight contest, and the lower outlier is from a children's eating contest.

The effect that will removing all outliers have on the mean and median of the data set is; C. median will remain unchanged and the mean will increase.

Learn more about mean and median here;

brainly.com/question/17060266

#SPJ4

Answer ASAP
Point R is at (3, 1.3) and Point T is at (3, 2.4) on a coordinate grid. The distance between the two points is
. (Input numbers and decimal point only, such as 8.2.) (3 points)

Answers

Answer:

1.1

Step-by-step explanation:

3 1/2 + 4 3/4 + 5 3/10

Answers

Answer: 13.55 or 13 11/20.

Step-by-step explanation: First, you have to find a common denominator, in this case, it would be 20. 2 can go into 20 10 times, 4 can go into 20 5 times, and 10 can go into 20 2 times. Then, you would NOW have 3 10/20 + 4 15/20 + 5 6/20, to have all the equal denominators. When you add, you would get 12 31/20, but now you have to simplify. Now, you SHOULD get 13 11/20 after simplifying. As a decimal, it would be 13.55 I hoped this helped!

FIND THE COTANGENT! Help me pls.

Answers

Answer: 5/8

Step-by-step explanation:

PLEASE ASAP 100 POITNS
a) Jill jumped 6 7/8 feet in the long-jump event. Jill’s best friend jumped 6 15/16 feet. How much farther did Jill’s best friend jump? Describe in words the process you used to solve the problem. (2 points)

b) If Jill’s best friend jumped farther than 6.5 feet, then she beat the school record and the seventh graders earn 50 points. If not, the eighth graders earn 50 points. Which grade should be awarded 50 points**? (1 point)

c) Claire and her partner, Grace, are throwing for the javelin event as a team. Claire threw the javelin 42 5/8 feet and Grace threw it 39 3/5 feet. If Claire and Grace combined their distances, what would their total feet thrown be? Show your work. (2 points)

d) If the total distance of Claire and Grace is greater than 83.7 feet, then the seventh graders earn 50 points. If not, then the eighth graders earn 50 points. Which grade should be awarded 50 points**? (1 point).

Answers

Answer:

Step-by-step explanation:

a) To find how much farther Jill's best friend jumped, we need to subtract Jill's distance from her best friend's distance. First, we need to convert both distances to the same unit, in this case inches. 6 7/8 feet is equal to 6 7/8 * 12 inches/foot = 82 7/8 inches. 6 15/16 feet is equal to 6 15/16 * 12 inches/foot = 83 16/16 inches. Now that both distances are in inches, we can subtract them: 83 16/16 inches - 82 7/8 inches = 1 9/16 inches. Therefore, Jill's best friend jumped 1 9/16 inches farther.

b) To determine if Jill's best friend beat the school record, we need to compare her distance to the record distance. The record distance for seventh graders is 6 feet 1 3/5 inches, which is equal to 6 1/5 * 12 inches/foot = 74 inches. Jill's best friend jumped 83 16/16 inches, which is greater than 74 inches. Therefore, Jill's best friend beat the school record and the seventh graders should be awarded 50 points.

c) To find the total distance thrown by Claire and Grace, we need to add their individual distances. First, we need to convert both distances to the same unit, in this case inches. 42 5/8 feet is equal to 42 5/8 * 12 inches/foot = 510 5/8 inches. 39 3/5 feet is equal to 39 3/5 * 12 inches/foot = 472 3/5 inches. Now that both distances are in inches, we can add them: 510 5/8 inches + 472 3/5 inches = 983 8/5 inches. Therefore, the total distance thrown by Claire and Grace is 983 8/5 inches.

d) To determine if the total distance thrown by Claire and Grace is greater than 83.7 feet, we need to compare their distance to the record distance. The record distance for seventh graders is 83.7 feet, which is equal to 83.7 * 12 inches/foot = 1004.4 inches. The total distance thrown by Claire and Grace is 983 8/5 inches, which is less than 1004.4 inches. Therefore, the total distance thrown by Claire and Grace is not greater than the record distance and the eighth graders should be awarded 50 points.

Jacob's body metabolizes caffeine at a rate of 13.5% per hour (so the amount of caffeine in Jacob's body decreases by 13.5% each hour) a. If Jacob consumes a cup of coffee with 96 mg of caffeine in it, how long will it take for Jacob's body to metabolize half of the 96 mg of caffeine? hours Preview b. If Jacob consumes an energy drink with 212 mg of caffeine in it, how long will it take for Jacob's body to metabolize half of the 212 mg of caffeine? hours Preview c. If Jacob consumes a cup of coffee with c mg of caffeine in it, how long will it take for Jacob's body to metabolize half of the cmg of caffeine? (Hint: your answer will be a numerical value.) hours Preview Submit

Answers

Jacob's body metabolizes caffeine at a rate of 13.5% per hour.

a)Jacob's body will takes 4.77 ~5 hours to metabolize half of the 96 mg of caffeine.

b) If Jacob consumes an energy drink with 212 mg of caffeine in it, Jacob's body will take 4.77948 ~ 5 hours to metabolize half of the 212 mg of caffeine.

c) Jacob consumes a cup of coffee with c mg of caffeine in it. Jacob's body will take 4.77948~ 5 hours to metabolize half of the c mg of caffeine.

What is decay rate?

The volume will slowly decrease at regular intervals and at a regular rate. This growth reduction is calculated using the exponential decay formula. The general form is y = a(1- r)ᵗ

We have, decay rate r = 13.5 % = 0.135

Initial value , a = 96 mg

plugging the value in formula we get,

y = 96(1 - 0.135)ᵗ --(1)

Now, Half of 96 is 48

so, 48 = 96(0.865)ᵗ --(2)

dividing equation by 96 we get

=> 48/96 = (0.865)ᵗ

taking natrual logarithm both sides,

=> ln(1/2) = ln ((0.865))

=> ln(1/2) = t In (0.865)

=> t = ln(1/2)/In (0.865)

=> t = 4.77948

Since, decay rate is constant so, half never changes for any .

a) 4.779~5 hours, long will it take for Jacob's body to metabolize half of the 96 mg of caffeine.

b) Jacob's body to metabolize half of the 212 mg of caffeine is 4.779~ 5 hours.

c) for Jacob's body to metabolize half of the cmg of caffeine is 4.77948.. So, we have 4.77948 ~ 5 hours is decay constant rate.

To learn more about Decay rate , refer:

https://brainly.com/question/27542728

#SPJ4

Y=x^3-4x^2-20x+48 use the rational zero theorem

Answers

The roots of the given polynomial using the rational zero theorem are;  2, -4 and 6.

How to use the rational zero theorem?

We are given the polynomial;

y = x³ - 4x² - 20x + 48

Since all coefficients are integers, we can apply the rational zeros theorem.

The trailing coefficient (the coefficient of the constant term) is 48

Find its factors (with the plus sign and the minus sign): ±1, ±2, ±3, ±4, ±6, ±8, ±12, ±16, ±24, ±48.

These are the possible values for p.

The leading coefficient (the coefficient of the term with the highest degree) is 1.

These are the possible rational roots:

±1, ±2, ±3, ±4, ±6, ±8, ±12, ±16, ±24, ±48.

Checking the possible roots: if a is a root of the polynomial P(x), the remainder from the division of P(x) by x - a should equal 0 (according to the remainder theorem, this means that P(a)=0

Plugging in those values, the only ones that yield P(a) = 0 are; 2, -4 and 6.

Thus, these are the roots of the given polynomial.

Read more about rational zero theorem at; https://brainly.com/question/17003818

#SPJ1

Math, help pls..................................

Answers

The measure of ∡ PRQ, if the sum of an interior angle of a triangle is  [tex]180^o[/tex] is  75°, so option B is correct.

What is the triangle?

One of the fundamental shapes in geometry is represented by the symbol Δ and consists of three connected vertices.

Given:

The lines m and n are parallel,

∡ QPR = 70°

As the lines are parallel, the alternate angles will be equal,

∡PQR  = ∡ QPN = 35°

The sum of the interior angles of the triangle is [tex]180^o[/tex]

∡ QPR  + ∡ PRQ + ∡ RQP = 180

70 + 35 + ∡ PRQ = 180

∡ PRQ = 180 - 105

∡ PRQ = 75°

Therefore, the measure of ∡ PRQ, if the sum of an interior angle of a triangle is [tex]180^o[/tex] is  75°.

To know more about Triangles:

https://brainly.com/question/16886469

#SPJ1

Work out the value of d in the equality below. 0.0043 = 4.3 × 10d​

Answers

Answer:

d= -3

Step-by-step explanation:

Rearrange variables to the left side of the equation: -4.3x 10d= -0.0043

Divide both sides of the equation by the coefficient of variable

Reduce the fraction

Convert fraction into negative exponential form

Convert both sides of the equation into terms with the same with same base

Simplify using exponent power rule

Based on the given conditions, corresponding exponents are equal

what is 13 -90
then 50/90=
then + together both answers and then multiply by 26 and what is the answer =

Answers

Answer: if there is an option for -1,989 then it would be that one

Step-by-step explanation:

13 - 90 would give you -77 and then 50/90 is would give you 0.5555555. -77 + 0.5 would be -76.5 which then you multiply by 26 gives you -1,989.

a line with a y-intercept of 4 passes through the point (14, -3). it also passes through point (x, -8). what is the x coordinate for that point?: * a) 8 b) 24 c) -24 d) 6 e) -6

Answers

The x-coordinate for the point  (x, -8), if Y-intercept is 4, and the coordinates of the point is (14, -3), is 24, so option B is correct.

What is line?

An object having an endless length and no width, depth, or curvature is called a line. Since lines can exist in two, three, or higher-dimensional environments, they are one-dimensional things.

Given:

Y-intercept = 4,

Coordinates of point =  (14, -3), (x, -8)

Calculate the line equation as shown below,

y = m x + 4

Here, m is the slope,

Put point (14, -3) in the equation,

-3 = m × 14 + 4

m = -7 / 14

m = -1 / 2

Hence, the equation of a line is,

y = -1/2 x + 4

Put point (x, -8),

-8 = -1/2 x + 4,

-12 = -1/2x

x = 24

To know more about the line:

https://brainly.com/question/2696693

#SPJ1

triangle p undergoes a sequence of transformations resulting in tringle q. which sequence of transformation could be used to show that triangle q is similar but not congruent to triangle P

Answers

The transformation could be used to show that triangle Q is similar but not congruent to triangle P is Dilation.

What is transformation?

A transformation is a general term for four specific ways to manipulate the shape and/or position of a point, a line, or geometric figure.

Given that, triangle P undergoes a transformations resulting in triangle Q.

We know, Dilation would not change the shape, just the size; the angle measures would be the same, and the ratio of corresponding sides would be equal to the scale factor used in the dilation. This would give us a similar, but not congruent, figure.

Hence, The transformation could be used to show that triangle Q is similar but not congruent to triangle P is Dilation.

For more references on transformations, click;

https://brainly.com/question/11709244

#SPJ1

consider two medical tests, a and b, for a virus. test a is 90% effective at recognizing the virus when it is present, but has a 5% false positive rate (indicating that the virus is present, when it is not). test b is 95% effective at recognizing the virus, but has a 10% false positive rate. the two tests use independent methods of identifying the virus. the virus is carried by 2% of all people.Say that a person is tested for the virus using only one of the tests, and that test comes back positive for carrying the virus.Which test returning positive is more indicative of someone really carrying the virus? Justify your answer mathematically (i.e. writing down your calculations).

Answers

We observe, that P(V|B)>P(V|A), so the person is more likely to have virus, however is still very small probability (only 15%), so in order to confirm illness, he should make one more test.

The area of mathematics known as probability deals with numerical representations of the likelihood that an event will occur or that a statement is true. An event's probability is a number between 0 and 1, where, roughly speaking, 0 denotes the event's impossibility and 1 denotes certainty.

P(V/A)  is not 0.95. It is opposite:

P(A/V)=0.95

From the text we can also conclude, that

P(A/∼V)=0.1

P(B/V)=0.9

P(B/∼V)=0.05

P(V)=0.01

P(∼V)=0.99

P(V/A) is not 0.95. It is opposite:

P(A/V)=0.95

From the text we can also conclude, that

P(A/∼V)=0.1

P(B/V)=0.9

P(B/∼V)=0.05

P(V)=0.01

P(∼V)=0.99

What we need to calculate and compare is P(V/A) and P(V/B)

P(V∩A)=P(A)⋅P(V/A)⇒P(V/A)=P(V∩A)/P(A)

P(V∩A) means, that person has a virus and it is detected, so

P(V∩A)=P(V)⋅P(A/V)

=0.01⋅0.95

=0.0095

P(A) is sum of two options: "Person has virus and it is detected" and "Person has no virus, but it was mistakenly detected", therefore:

P(A)=P(V)⋅P(A/V)+P(∼V)⋅P(A/∼V)=0.01⋅0.95+0.99⋅0.1=0.1085

Dividing those two numbers we obtain

P(V/A)=0.0095/0.1085=0.08755760368663594

Analogically,

P(V/B)=P(V∩B)/P(B)=P(V)⋅P(B/V)/P(V)⋅P(B/V)+P(∼V)⋅P(B/∼V)

=0.01⋅0.9/0.01⋅0.9+0.99⋅0.1

=0.1538461538461539

We see that P(V|B)>P(V|A), meaning the individual is more likely to have a virus, but is still very unlikely (only 15%), therefore person should perform another test to confirm illness.

To learn more about probability visit: brainly.com/question/11234923

#SPJ4

for cos(theta) = -3/5, and pi/2 < theta < pi, find the following trig values

sin theta
tan theta
csc theta
sec theta
cot theta

Answers

For the given range of theta, cos(theta) is negative, which means that theta is in the third or fourth quadrant. In these quadrants, the sine function is positive, the tangent function is negative, and the cotangent function is negative.

Since cos(theta) = -3/5, we can use the Pythagorean identity to find the value of sin(theta):

sin^2(theta) + cos^2(theta) = 1

sin^2(theta) = 1 - cos^2(theta)

sin^2(theta) = 1 - (-3/5)^2

sin^2(theta) = 1 - 9/25

sin^2(theta) = 16/25

sin(theta) = sqrt(16/25) = 4/5

Therefore, the value of sin(theta) is 4/5.

We can also use the identity cot(theta) = 1/tan(theta) to find the value of cot(theta):

cot(theta) = 1/tan(theta)

cot(theta) = 1/(sin(theta)/cos(theta))

cot(theta) = cos(theta)/sin(theta)

cot(theta) = (-3/5)/(4/5)

cot(theta) = -3/4

Therefore, the value of cot(theta) is -3/4.

The values of the other trigonometric functions can be found using the definitions of these functions:

tan(theta) = sin(theta)/cos(theta) = (4/5)/(-3/5) = -4/3

csc(theta) = 1/sin(theta) = 1/(4/5) = 5/4

sec(theta) = 1/cos(theta) = 1/(-3/5) = -5/3

Therefore, the values of the trigonometric functions for the given range of theta are:

sin(theta) = 4/5

tan(theta) = -4/3

csc(theta) = 5/4

sec(theta) = -5/3

cot(theta) = -3/4

Solve
16) -x<-x+7(x-2)

Answers

The solution of the given inequality -x < -x + 7 ( x - 2 ) is given by x > 2.

As given in the question,

Inequality is given by :

-x < -x + 7 ( x - 2 )

Open the parenthesis of the given inequality we have,

⇒ -x < -x + 7x - 14

Collect all the like terms on the same side we get,

⇒ - x < 6x  - 14

Subtract 6x from both the side of the inequality we get,

⇒ - x - 6x < 6x - 6x - 14

⇒ -7x < -14

Divide both the side of the inequality by -7 we get,

⇒ -7x/ -7 > -14 / -7

⇒ x > 2

x has all the value greater than 2.

Therefore, for the given inequality the solution is given by x > 2.

Learn more about inequality here

brainly.com/question/28823603

#SPJ4

Simplify for all questions.

Answers

Answer:

3y; -2a; -5y; 3c; -6

-----------------------------------------

Question 1[tex]-\sqrt{9y^2}=-(-3y)= 3y, \ since\ y < 0[/tex]

Question 2[tex]0.5\sqrt{16a^2} =0.5(-4a)=-2a,\ since\ a < 0[/tex]

Question 3[tex]-5\sqrt{y^2} =-5y, since \ y > 0[/tex]

Question 4[tex]3\sqrt{c^2} =3c,\ since \ c \geq 0[/tex]

Question 5[tex]\sqrt{(3-\sqrt{15})^2 } -\sqrt{(3+\sqrt{15})^2 } =-(3-\sqrt{15}) -(3+\sqrt{15})=-6, since\ 3 < \sqrt{15}[/tex]

Find all solutions of the equation 2sinx +√3 = 0

The answer is A+Bkπ and C+Dkπ where k is any integer, 0

Answers

Answer:

Step-by-step explanation:

2sin x=-√3

[tex]sin~x=-\frac{\sqrt{3} }{2} =-sin~\frac{\pi }{3} =sin (\pi +\frac{\pi }{3} ),sin(2\pi -\frac{\pi }{3} )\\=sin(\frac{4\pi }{3} +2k\pi ),sin (\frac{5\pi }{3} +2k\pi )\\=sin(A+Bk\pi ),sin(C+Dk\pi )\\x=A+Bk\pi ,C+Dk\pi \\where~A=\frac{4\pi }{3} ,B=2\\C=\frac{5\pi }{3} ,D=2[/tex]

Find axis of symmetry

Answers

Answer:

Step-by-step explanation:

x=-4

Identify the factors in the expression 3(m + 2).
A. 3 and (m + 2)
B. m and 2
OC. 3 and m
D. 3 and 2

Answers

The factors of the expression 3(m + 2) are 3 and (m + 2) and the correct option is A. 3 and (m + 2).

What is a factor

A factor in an expression of number, variable, term or any other longer expression that is multiplied by another. In mathematics, a number or algebraic expression that divides another number or expression evenly without remainder is also called a factor.

The question have an algebraic expression; 3(m + 2)

considering the four options;

A. 3 and (m + 2)

B. m and 2

C. 3 and m

D. 3 and 2

Only option A. with 3 and (m + 2) can be said to be factors as multiplying them will give 3(m + 2) and can both divide the expression 3(m + 2) evenly without a remainder.

In conclusion, 3 and (m + 2) are factors for the given expression and option A. 3 and (m + 2) is correct.

Learn more about factors here:https://brainly.com/question/25829061

#SPJ1

Other Questions
To allow two pieces of copper tubing to be joined by soldering or brazing without any fitting, which of the following must be used?A. Fitting endB. Sweat treeC. Swage jointD. Solder end since the early 1990s, school pe programs in the united states have focused on pe, which emphasizes public health objectives and the health benefits of physical activity. explain why the corrosion rate for a small anode-to-cathode area will be higher than for a large anode-to-cathode area a school board has a plan to increase participation in the pta. currently only about 16 parents attend meetings. suppose the school board plan results in logistic growth of attendance. the school board believes their plan can eventually lead to an attendance level of 48 parents. in the absence of limiting factors the school board believes its plan can increase participation by 30% each month. let m denote the number of months since the participation plan was put in place, and let p be the number of parents attending pta meetings. while enrolled in a weight-loss program for employees of the company where he worked, gavin lost nearly 100 pounds over a period of 14 months. at the same time, his bmr slowed. then he was laid off. over the next six months, his bmr remained slowed, and he regained two-thirds of the weight he had lost. which of the following hypotheses best explains this phenomenon? What were the major causes of the War of 1812?disputes over navigable waters and an interest in increasing tradedesire to gain more territory and recapture land lost during the Revolutionimpressment of American sailors, resentment over the Revolution, and trade restrictionsU.S. effort to extend the northern border into Canada and an opportunity to support Native Americans The best way for a monopoly to maximize profits is to charge one very high price located at the upper left-hand side of their demand curve. Sticking to this fixed price is the best pricing strategy for the monopoly in order to maximize profits and capture all individuals on their demand curve.True or False? mrs. chen has set up different stations in her classroom for the children's lessons. each station emphasizes a different way of learning such as music, body movements, and interpersonal exercises. this approach represents the application of group of answer choices sternberg's triarchic theory of intelligence. gardner's theory of multiple intelligences. kauffman's assessment battery. wechsler's intelligence scale. The delegates to the Constitutional Convention agreed to establish a republic. Which of these describes a republic?a. Government led by a Kingb. Government led by a small, group of powerful peoplec. Government led by elected officialsd. Government led by a military. if a stock consistently goes down (up) by 1.37% when the market portfolio goes down (up) by 1.08%, then its beta equals: you are designing an elevator for a hospital. the force exerted on a passenger by the floor of the elevator is not to exceed 1.50 times the passenger's weight. the elevator accelerates upward with constant acceleration for a distance of 2.8 m and then starts to slow down. What is the maximum speed of the elevator? you are working as a network engineer for an isp. you have successfully set up a home internet connection for a client. the isp requires the network engineers to provide a document with specifications and tests that the customer can run on their own to check various aspects of their connectivity. however, in the document that you provided you forgot to mention the functionality of speedtest.net in the document. so, the client calls you with a query about the use of speedtest.net. which of the following will be your reply to the client in this scenario? It is used for bandwidth speed testsIt is used to identify Internet access.It is used to test the router's stability.It is used for broadband cable tests. 02.01 Impact of Colonization I need help maxxam legally owned the land which it was cutting and on which julia staged her protest. her actions broke the law and cost the company millions of dollars. were her actions justified? in an experiment with patients, a group of researchers used nonlinguistic hierarchical shape stimuli such as the example here. how did the results differ from those obtained with hierarchical letter stimuli? 4. Given the equationy =- 3x + 2.5:a) When x is 1, what value of y makes the equation true? identify the method: this method is run before the file system is mounted and made available (assumes that no other file-system activity is on-going while it runs); once finished, the on disk file system should be consistent and thus can be made accessible to users. select one: a. journaling b. backpointer-based consistency c. fsck d. superblock Exhalation begins whenA. inspiratory muscles relax.B. diaphragm contracts.C. blood circulation is the lowest.D. both A and BE. all of the above only the adult adoptee can decide if s/he may access the health information of his or her biological parents for health risk purposes? what was the cause of the battle of lake erie